Spooky action at a distance and various interpretations of QT

In summary, according to my analysis, if the photons were purely classical (i.e. not entangled) both detectors would fire 12½% of the time but since the two photons are entangled and their planes of polarisation are at right angles, QT predicts that the two detectors will never fire together. This effect may be explained by a simplistic interpretation of QT in which when the two photons leave the source they are in a superposition of an infinite number of states with their polarisation planes in all possible directions (but always at right angles to each other); when photon A reaches P1 a 'measurement' of its plane of polarisation is made and the wavefunction partially collapses into just two possible states
  • #36
J O Linton said:
I was under the impression that the Pilot Wave interpretation assumed that all the entities involved had definite properties at all times
yes, I omitted a disclaimer that I usually make: “with apologies to the Bohmians” - didn’t seem necessary in this context
 
  • Haha
Likes gentzen and mattt
Physics news on Phys.org
  • #37
J O Linton said:
I was under the impression that the Pilot Wave interpretation assumed that all the entities involved had definite properties at all times
The Pilot Wave does not say that all properties always have definite values, only that position always does. In fact, the Pilot Wave interpretation, strictly speaking, says that position is the only property that anything ever has; measurements that we think of as measuring something other than position are actually measuring position, we are just interpreting them as measuring something else.

For example, the result of what we think of as a polarization measurement of a photon actually depends, in this interpretation, on the position of the photon as it enters the polarizer; one set of positions results in the photon going through the polarizer, and the remaining positions result in the photon getting absorbed. We interpret this as the polarization of the photon being either parallel to or perpendicular to the orientation of the polarizer, but actually, according to the Pilot Wave interpretation, there is no such thing as "polarization" as a separate property; all we're actually measuring is the photon's position.
 
  • Like
Likes Demystifier and mattt
  • #38
So what, exactly is the Bomian (Pilot Wave?) interpretation in this context. Does it imply some sort of SAAAD?
 
  • #39
J O Linton said:
So what, exactly is the Bomian (Pilot Wave?) interpretation in this context. Does it imply some sort of SAAAD?
The Pilot Wave interpretation is fully deterministic, so the results of all experiments that are ever done through the entire history of the universe are determined by the initial conditions. (Strictly speaking, this would be called "superdeterminism" since the initial conditions would also have to fix all of the experimental settings that will ever be made, such as the orientations of polarizers in photon polarization measurements.) A fully deterministic theory wouldn't normally be thought of as having any SAAAD.

However, the usual formulation of the Pilot Wave interpretation is non-relativistic, which means it does not limit signal speeds to the speed of light, and one of the criteria for SAAAD is usually taken to be the possibility of signals being sent faster than light. There have been some attempts to formulate a relativistic version of the Pilot Wave interpretation, but none of them have gained general acceptance.
 
  • #40
I can't say that I like the sound of that!

Anyway, now that we have discussed some of the various interpretations and removed a few of the misconceptions which plague my understanding of the subject, I would beg you to consider a slight modification to my experimental setup.

A third polaroid, P3, at 45 degrees to the vertical is interposed between the source and P1.

Firstly - what is the prediction of QM as regards the possibility that the two detectors fire together? Does the passage of photon A through P3 'break the entanglement' - in which case we now have two classical photons traveling at +-45 deg towards two vertical polarizers with the possibility that both detectors fire; or is it the case that if photon A happens to pass through both P3 and P1, then photon B is still prevented from passing through P2? If so, how is this explained using the interpretations we have discussed?
 
  • #41
J O Linton said:
I would beg you to consider a slight modification to my experimental setup.A third polaroid, P3, at 45 degrees to the vertical is interposed between the source and P1.
Now we're back to a question about what quantum mechanics says will happen with particular configuration of polarizers and detectors - the answer will of course be the same no matter what interpretation you choose. But before we go there, two things:
1) In your first post you said that you expected both detectors to trigger in 12.5% of your tests runs when the two photons are not entangled and 0% if they are. Where did that 12.5% come from? If I'm understanding your description properly it should be 25% in the non-entangled case: 50% probability that A passes P1, independent 50% probability that B passes P2 gives us a 25% probability that they both pass. (I do agree about the 0% for the entangled case).
2) You went out of your way to specify that P1 is closer to the source than P2. If you're doing this so that you can think in terms of the interaction with P1 happening first... Don't. The two interaction are space-like separated so there's no way of saying which one is first. If you don't get the same result no matter which photon first encounters a polarizer, you're doing something wrong.

And with that said... What do you expect to happen in your new configuration, for both the entangled and the unentangled case?
 
  • Like
Likes mattt
  • #42
Nugatory said:
That is not right, and may be the source of your confusion here. When the photon is emitted there is one world in which the plane of polarization has no value. When the polarization is measured this world splits into two, one in which the photon passed through the polarizer and one in which it didn’t. In the world in which the photon passed through the polarizer, the entangled partner will not pass through a polarizer at the same angle if we try.

You are reasoning as if the polarization has some definite value even though it hasn’t been measured. That doesn’t work in any interpretation.
I would very much like to know if there is an interpretation of QM as attributed at MWI by Linton in #32, where the world splits not on the measurements, but on the creation of particles when a symmetry would need to be broken, and where ulterior measurements only "select" one subset of this many worlds (that would be the collapse of the wave function) ? Or is it just rephrasing the standard interpretation ?
 
  • #43
Nugatory said:
Now we're back to a question about what quantum mechanics says will happen with particular configuration of polarizers and detectors - the answer will of course be the same no matter what interpretation you choose. But before we go there, two things:
1) In your first post you said that you expected both detectors to trigger in 12.5% of your tests runs when the two photons are not entangled and 0% if they are. Where did that 12.5% come from? If I'm understanding your description properly it should be 25% in the non-entangled case: 50% probability that A passes P1, independent 50% probability that B passes P2 gives us a 25% probability that they both pass. (I do agree about the 0% for the entangled case).
2) You went out of your way to specify that P1 is closer to the source than P2. If you're doing this so that you can think in terms of the interaction with P1 happening first... Don't. The two interaction are space-like separated so there's no way of saying which one is first. If you don't get the same result no matter which photon first encounters a polarizer, you're doing something wrong.

And with that said... What do you expect to happen in your new configuration, for both the entangled and the unentangled case?
The 12.5% comes about because you must integrate across all possible values of the initial polarisation. This means writing down an expression for the probability of coincidence for every angle and integrating it. Of course I am assuming that the two photons are oriented at right angles, even in the 'classical' case.

I agree with you that the position of the second detector is irrelevant. I just wanted to specify the experiment in such a way as to forestall any possible discussion of this issue.

As regards the outcome of my modified experiment, I have never studied QM and am not qualified to do the necessary calculations so I genuinely don't know the answer and am very interested to be told what it is. For what it is worth I am 90% confident that the 45 degree polarizer will not break the entanglement and that the two detectors will still not fire together buit, as with so many aspects of QT, I could be wrong.
 
  • #44
J O Linton said:
Of course I am assuming that the two photons are oriented at right angles, even in the 'classical' case.
Ok, I had not picked that up from the original post.
For what it is worth I am 90% confident that the 45 degree polarizer will not break the entanglement
You are mistaken. Just like the original vertically oriented one, that polarizer is measuring the polarization so ends the entanglement.
 
  • #45
I am not so sure. I suspect that the action of simply passing a photon through a polarizer does not in itself constitute a measurement but that decoherence must occur as well. In other words, the measurement is only actually made when the photon is recorded by D1 or D2.

The point is absolutely crucial and it was actually to test this very point that I started this whole thread. Are you qualified in QM and have you actually calculated the result using the standard Bra-Ket rules? If not, I will, respectfully, reserve judgment until I get a reply from someone who has the appropriate skills.
 
  • #46
J O Linton said:
IAre you qualified in QM and have you actually calculated the result using the standard Bra-Ket rules?
yes.
 
  • #47
Excellent! Now we can really start to sort things out!

Am I right in saying that after photon A has either passed through or been absorbed by P3, there are now two equally probable possiblilities:
1. Photon A passes through through P3 and we now have two entirely classical, unentangled photons traveling towards P1 and P2, one at +45 degrees and the other at -45 degrees. The probability that both pass through P1 and P2 is therefore 25%.
2. Photon A is absorbed by P3 and there is zero probability that both detectors will fire.
The conclusion is that both detectors will fire 12.5% of the time.

Under the MWI, when A reaches P3 the original world in which the polarization of the two photons was indeterminate splits into two worlds which can no longer interfere with each other (because you say that the entanglement is broken). In one world photon A passes through P3 and in the other it is absorbed. When B reaches P2 a further split into two worlds occurs making 4 possible worlds (though not with equal probabilities). Is this correct?
 
  • #48
Christian Thom said:
I would very much like to know if there is an interpretation of QM as attributed at MWI by Linton in #32, where the world splits not on the measurements
The reason the MWI has worlds splitting on measurements is that measurements are when decoherence happens.

Christian Thom said:
, but on the creation of particles when a symmetry would need to be broken
QM does not say that decoherence happens on "the creation of particles when a symmetry would need to be broken", so no, there is no such interpretation.

Christian Thom said:
Or is it just rephrasing the standard interpretation ?
It's not describing any intepretation that I'm aware of.
 
  • #49
Nugatory said:
that polarizer is measuring the polarization so ends the entanglement.
Note that under some interpretations, such as the MWI, measurements do not end entanglement; they only extend it to the measuring devices (and ultimately to the environment through decoherence).
 
  • #50
J O Linton said:
I suspect that the action of simply passing a photon through a polarizer does not in itself constitute a measurement but that decoherence must occur as well.
Decoherence does occur when the photon passes through the polarizer, because the polarizer might absorb the photon. Those two alternatives (passes through the polarizer vs. absorbed by the polarizer) can never interfere with each other, so they are decohered. The detector after the polarizer in this case just allows us humans to detect which alternative happened (or, in interpretations like the MWI, happened in our branch of the wave function).

If you want a device that can act on photons without decoherence, you need something like a beam splitter, where both alternatives pass through the splitter, they just come out in different directions. In that case, decoherence would not occur until a photon reached a detector, and you could make the two alternatives interfere by directing them into another beam splitter (as in, for example, a Mach-Zehnder interferometer).

J O Linton said:
The point is absolutely crucial and it was actually to test this very point that I started this whole thread.
In other words, we have spent 50 posts now going round and round about something that could have been answered in one post (I just answered it in this one), if you had just asked the question you actually wanted answered in the OP of this thread. Please bear that in mind for future discussions: if you want an answer to a particular question, just ask the question. Don't try to construct an elaborate scenario that you think will lead to an answer to the question; it might not, because you might not understand the subject well enough.
 
  • Like
Likes hutchphd
  • #51
PeterDonis said:
The reason the MWI has worlds splitting on measurements is that measurements are when decoherence happens.QM does not say that decoherence happens on "the creation of particles when a symmetry would need to be broken", so no, there is no such interpretation.It's not describing any intepretation that I'm aware of.
Thank you for your answer.
 
  • #52
J O Linton said:
I am not so sure. I suspect that the action of simply passing a photon through a polarizer does not in itself constitute a measurement but that decoherence must occur as well. In other words, the measurement is only actually made when the photon is recorded by D1 or D2.

The point is absolutely crucial and it was actually to test this very point that I started this whole thread. Are you qualified in QM and have you actually calculated the result using the standard Bra-Ket rules? If not, I will, respectfully, reserve judgment until I get a reply from someone who has the appropriate skills.
I would think it depends on the polarizer : if the other polarization is absorbed, it might count as a measurement. If the other polarization is reflected, it could be used to reconstruct the original wave so it is not a measurement IMHO.
 
  • #53
Christian Thom said:
I would think it depends on the polarizer : if the other polarization is absorbed, it might count as a measurement.
It does. See my post #50. My understanding is that this is the type of polarizer the OP was using.

Christian Thom said:
If the other polarization is reflected, it could be used to reconstruct the original wave so it is not a measurement IMHO.
I think the more usual term for this type of device is "polarizing beam splitter".
 
  • #54
I think, PeterDonis, you are being a little unfair on me. The 50 posts have already cleared up several misconceptions of mine which might have been missed if I had inroduced the modified experiment at the start.
But thank you all the same for confirming Nugatories statement that the polarizer does break the entanglement. And since you have not contradicted anything which I said in #47, I take it that what I said there is basically correct.

My reason for deferring the introduction of P3 was that, assuming that P3 did not break the entanglement, the case for some sort of SAAAD between the two photons would be quite strong. But since the entanglement is broken there is no need to consider this possibility any more.
 
  • #55
In view of what you said in #53, if P3 was a polarizing beam splittier and the other polarization was allowed to go off into space - are you saying that the entanglement would remain and that the detectors would never fire together?
 
  • #56
J O Linton said:
The 50 posts have already cleared up several misconceptions of mine which might have been missed if I had inroduced the modified experiment at the start.
I'm not saying you should have introduced some "modified experiment" (not sure what you mean by this) at the start. I'm saying that, since the primary question you apparently wanted an answer to (going by the post of yours that I quoted) is "does decoherence occur immediately when a photon encounters a polarizer, or only when it encounters a detector after passing through a polarizer?", you could have just asked that question in your OP. You would have gotten an answer in one post, and you could have followed up with other questions with that primary point correctly understood at post #2 instead of post #50. I'm glad we have still cleared up some misconceptions of yours, but that could have been done with the thread started the way I describe too, since your follow-up questions would presumably still have included other things you asked about in this thread.
 
  • #57
J O Linton said:
thank you all the same for confirming Nugatories statement that the polarizer does break the entanglement.
I didn't say the polarizer breaks the entanglement. I said it causes decoherence. That's not the same thing. See my post #49 in response to @Nugatory.
 
  • #58
So what is your response to #55?
 
  • #59
J O Linton said:
since you have not contradicted anything which I said in #47, I take it that what I said there is basically correct.
As far as the MWI part of post #47 is concerned, you need to rethink that in the light of my post #49.

The first partof post #47 looks correct to me as far as the calculation of probabilities is concerned, although some of your description in words is interpretation dependent.
 
  • #60
J O Linton said:
So what is your response to #55?
It has the same confusion between "breaking entanglement" and decoherence that I pointed out in post #57.
 
  • #61
I admit that I am totally confused as to the difference berween 'breaking entanglement' and 'decoherence' but you haven't answered my question - if P3 is a beam splitting polarizer, will that make any difference to the behaviour of the detectors and if not, why not?
 
  • #62
J O Linton said:
if P3 was a polarizing beam splittier and the other polarization was allowed to go off into space
If the other polarization is just allowed to escape, the experiment is the same as if P3 were an ordinary polarizer that only let through the polarization that is going to encounter P1. To make any difference, the other polarization would have to have something done with it within the lab--for example, put another detector in that beam, or reflect it with a mirror so it can potentially interfere with one of the other beams.
 
  • #63
I find that simply incredible. What you are saying is that you can prevent the two detectors D1 and D2 from firing together by 'doing something' with a potential beam that is going somewhere completely different.
 
  • #64
PeterDonis said:
since the primary question you apparently wanted an answer to (going by the post of yours that I quoted) is "does decoherence occur immediately when a photon encounters a polarizer, or only when it encounters a detector after passing through a polarizer?", you could have just asked that question in your OP. You would have gotten an answer in one post,
The trouble is - if you don't understand a topic very well, you don't know what questions to ask!

Thanks anyway.
 
  • #65
J O Linton said:
What you are saying is that you can prevent the two detectors D1 and D2 from firing together by 'doing something' with a potential beam that is going somewhere completely different.
In some runs of the experiment, yes. Suppose we put a detector D3 in the path of P3. Then on any run where D3 fires, D1 will not, so D1 and D2 can never fire together on any run where D3 fires.

On a run where D3 does not fire, the situation is the same as the case where P3 is just an ordinary polarizer and allows the photon to pass through and encounter P1. In that case, the probability of D1 and D2 firing together is what was previously calculated for that case.
 
  • #66
J O Linton said:
if you don't understand a topic very well, you don't know what questions to ask!
You apparently knew when you started this thread that "does decoherence occur immediately when a photon encounters a polarizer, or only when it encounters a detector after passing through a polarizer?" was the primary question you wanted the answer to. So you could have just asked it.

The fact that you wouldn't have known at the time that that was a "good" question to ask is beside the point, because the rule I'm suggesting that you follow is not "only ask questions that you know are good ones". It's "ask the primary question you want to know the answer to directly, instead of trying to ask it indirectly by making up a scenario that you think will lead to an answer to it". The fact that you don't understand a topic very well and so aren't sure what questions to ask is exactly why you should ask the questions you want answers to directly: because if you don't understand a topic very well, you aren't going to be very good at asking a question indirectly by making up a scenario, and a lot more time will be spent just trying to get to your actual question. In this case, about 50 posts more.
 
  • Like
Likes phinds
  • #67
J O Linton said:
I admit that I am totally confused as to the difference berween 'breaking entanglement' and 'decoherence'
"Breaking entanglement" is interpretation dependent. For example, if P3 absorbs a photon, that "breaks entanglement" in a collapse interpretation, but not in the MWI.

Decoherence, however, is not interpretation dependent. The two alternatives "P3 absorbs the photon" and "P3 passes the photon" are decoherent in any interpretation.
 
  • #68
PeterDonis said:
The reason the MWI has worlds splitting on measurements is that measurements are when decoherence happens.QM does not say that decoherence happens on "the creation of particles when a symmetry would need to be broken", so no, there is no such interpretation.It's not describing any intepretation that I'm aware of.
After some search and the viewing of a very good recent video from S. Hossenfelder, I think it is close to the super-determinism, as it exploits the same loophole of Bell's theorem : the statistical independence.
 
  • Skeptical
Likes weirdoguy
  • #69
Christian Thom said:
a very good recent video from S. Hossenfelder, I think it is close to the super-determinism
That's pretty much the viewpoint Hossenfelder seems to hold, yes.
 
  • #70
Christian Thom said:
After some search and the viewing of a very good recent video from S. Hossenfelder, I think it is close to the super-determinism, as it exploits the same loophole of Bell's theorem : the statistical independence.

I'm not sure what to think about her video. She sort of ignored issue that most people I've seen have presented as the deal breaker in favoring super-determinism. According to others, in order for super-determinism to be plausible, the past or initial conditions of the universe would have to be very very "special", so as to be in just the right way that we happen to see the correlations we do. I'm careful not to validate these concerns, because I haven't studied it closely enough. I may be one of the people she talks about who doesn't know what they're talking about.

There is apparently a similar issue with MWI, in that in some of the less common world lines, people will observe universes that look highly "special", in various weird ways, or look like some seriously spooky action at a distance was happening.

Sabine is hoping that in the future, large scale data analysis, using AI, and experiments of events in the non-chaotic regime, will make it look obvious that our reality is super-deterministic. But if that happens, couldn't we also just be equally a very very special world in a MWI reality at that point?

A truly randomly generated number will have equally likely outcomes, each time a new one is generated. Suppose a uniform random number generator generates numbers between 1 and 10^100^100^100. It's theoretically possible that you could generate the same number 10^100 times in a row, or even indefinitely. In fact that result is just as likely as any other sequences of numbers. It's just not typical in the sense that most possible sequences will look more "random" (or more typical).

In fact, we don't need MWI, or super-determinism, or entanglement at all right? We can just have true randomness, and be really really lucky? In any case all of these possibilities (being really lucky rolling the dice, a very very special and precisely and elaborately arranged domino arrangement, or we're a special highly atypical world in a MWI universe) seem unappealing for similar reasons. We usually rely on the assumption that if QM is giving us randomness, it will look like "randomness", rather than we just might get super lucky. Or we assume that even if MWI is correct, that the world line we've experienced thus far is fairly typical.

This said, maybe the "fine tuning" problem in QM foundations isn't what people make it out to be. Maybe the narrowest sense in which the independence needs to be violated need not be "
"special" in the sense people make out it be?
 
Last edited:
  • Like
Likes DrChinese, Christian Thom and gentzen

Similar threads

  • Quantum Interpretations and Foundations
Replies
31
Views
2K
  • Quantum Interpretations and Foundations
2
Replies
52
Views
1K
  • Quantum Interpretations and Foundations
Replies
15
Views
338
  • Quantum Interpretations and Foundations
Replies
21
Views
3K
  • Quantum Interpretations and Foundations
3
Replies
79
Views
5K
  • Quantum Interpretations and Foundations
2
Replies
54
Views
4K
  • Quantum Interpretations and Foundations
4
Replies
138
Views
5K
  • Quantum Interpretations and Foundations
Replies
14
Views
2K
  • Quantum Interpretations and Foundations
2
Replies
38
Views
4K
  • Quantum Physics
Replies
13
Views
693
Back
Top